Are there further primes of the form $varphi(n)^{varphi(varphi(n))}+1$?












9












$begingroup$


For positive integers $n$ , define $$f(n):=varphi(n)^{varphi(varphi(n))}+1$$ where $varphi(n)$ denotes the totient function.



According to my calculation, for the following positive integers $n$ , $f(n)$ is a prime number : $$[1, 2, 3, 4, 5, 6, 7, 8, 9, 10, 12, 14, 18, 97, 119, 153, 194, 195, 208, 224, 23
8, 260, 280, 288, 306, 312, 336, 360, 390, 420]$$ and upto $n=10^4$, no further prime occurs. For $n>6$ , we have $varphi(varphi(n))>1$ and $varphi(n)>1$ hence $varphi(varphi(n))$ must be a power of $2$. The number is then a generalized Fermat-number.




Do further primes $f(n)$ exist ?











share|cite|improve this question









$endgroup$








  • 1




    $begingroup$
    @paw88789 To get a prime , this must be the case.
    $endgroup$
    – Peter
    Jul 26 '18 at 16:22






  • 2




    $begingroup$
    @Joffan if $q$ is an odd prime factor of $n$ , then $a^n+1$ is divisible by $a^{n/q}+1$
    $endgroup$
    – Peter
    Jul 26 '18 at 16:24






  • 1




    $begingroup$
    Have you looked it up in the OEIS? I just did. No relevant results.
    $endgroup$
    – Robert Soupe
    Jul 26 '18 at 18:19






  • 1




    $begingroup$
    Then I tried Select[Range[1000], PrimeQ[EulerPhi[#]^EulerPhi[EulerPhi[#]] + 1] &] in Wolfram Alpha. I probably need Mathematica if I want to confirm your assertion up to $10^4$.
    $endgroup$
    – Robert Soupe
    Jul 26 '18 at 18:22






  • 1




    $begingroup$
    @RobertSoupe : Pari/GP is a free software which is able to deal with that problem nicely. Note btw, that many $n$ have the same value $varphi(n)$ and thus the same $text{isprime}(f(n)$ -result, so time consumption could be much reduced when you avoid multiple computation of the $f(n)$ at different $n$ leading to the same result.
    $endgroup$
    – Gottfried Helms
    Jul 26 '18 at 20:58
















9












$begingroup$


For positive integers $n$ , define $$f(n):=varphi(n)^{varphi(varphi(n))}+1$$ where $varphi(n)$ denotes the totient function.



According to my calculation, for the following positive integers $n$ , $f(n)$ is a prime number : $$[1, 2, 3, 4, 5, 6, 7, 8, 9, 10, 12, 14, 18, 97, 119, 153, 194, 195, 208, 224, 23
8, 260, 280, 288, 306, 312, 336, 360, 390, 420]$$ and upto $n=10^4$, no further prime occurs. For $n>6$ , we have $varphi(varphi(n))>1$ and $varphi(n)>1$ hence $varphi(varphi(n))$ must be a power of $2$. The number is then a generalized Fermat-number.




Do further primes $f(n)$ exist ?











share|cite|improve this question









$endgroup$








  • 1




    $begingroup$
    @paw88789 To get a prime , this must be the case.
    $endgroup$
    – Peter
    Jul 26 '18 at 16:22






  • 2




    $begingroup$
    @Joffan if $q$ is an odd prime factor of $n$ , then $a^n+1$ is divisible by $a^{n/q}+1$
    $endgroup$
    – Peter
    Jul 26 '18 at 16:24






  • 1




    $begingroup$
    Have you looked it up in the OEIS? I just did. No relevant results.
    $endgroup$
    – Robert Soupe
    Jul 26 '18 at 18:19






  • 1




    $begingroup$
    Then I tried Select[Range[1000], PrimeQ[EulerPhi[#]^EulerPhi[EulerPhi[#]] + 1] &] in Wolfram Alpha. I probably need Mathematica if I want to confirm your assertion up to $10^4$.
    $endgroup$
    – Robert Soupe
    Jul 26 '18 at 18:22






  • 1




    $begingroup$
    @RobertSoupe : Pari/GP is a free software which is able to deal with that problem nicely. Note btw, that many $n$ have the same value $varphi(n)$ and thus the same $text{isprime}(f(n)$ -result, so time consumption could be much reduced when you avoid multiple computation of the $f(n)$ at different $n$ leading to the same result.
    $endgroup$
    – Gottfried Helms
    Jul 26 '18 at 20:58














9












9








9


3



$begingroup$


For positive integers $n$ , define $$f(n):=varphi(n)^{varphi(varphi(n))}+1$$ where $varphi(n)$ denotes the totient function.



According to my calculation, for the following positive integers $n$ , $f(n)$ is a prime number : $$[1, 2, 3, 4, 5, 6, 7, 8, 9, 10, 12, 14, 18, 97, 119, 153, 194, 195, 208, 224, 23
8, 260, 280, 288, 306, 312, 336, 360, 390, 420]$$ and upto $n=10^4$, no further prime occurs. For $n>6$ , we have $varphi(varphi(n))>1$ and $varphi(n)>1$ hence $varphi(varphi(n))$ must be a power of $2$. The number is then a generalized Fermat-number.




Do further primes $f(n)$ exist ?











share|cite|improve this question









$endgroup$




For positive integers $n$ , define $$f(n):=varphi(n)^{varphi(varphi(n))}+1$$ where $varphi(n)$ denotes the totient function.



According to my calculation, for the following positive integers $n$ , $f(n)$ is a prime number : $$[1, 2, 3, 4, 5, 6, 7, 8, 9, 10, 12, 14, 18, 97, 119, 153, 194, 195, 208, 224, 23
8, 260, 280, 288, 306, 312, 336, 360, 390, 420]$$ and upto $n=10^4$, no further prime occurs. For $n>6$ , we have $varphi(varphi(n))>1$ and $varphi(n)>1$ hence $varphi(varphi(n))$ must be a power of $2$. The number is then a generalized Fermat-number.




Do further primes $f(n)$ exist ?








number-theory elementary-number-theory prime-numbers totient-function






share|cite|improve this question













share|cite|improve this question











share|cite|improve this question




share|cite|improve this question










asked Jul 26 '18 at 16:06









PeterPeter

47.3k1039128




47.3k1039128








  • 1




    $begingroup$
    @paw88789 To get a prime , this must be the case.
    $endgroup$
    – Peter
    Jul 26 '18 at 16:22






  • 2




    $begingroup$
    @Joffan if $q$ is an odd prime factor of $n$ , then $a^n+1$ is divisible by $a^{n/q}+1$
    $endgroup$
    – Peter
    Jul 26 '18 at 16:24






  • 1




    $begingroup$
    Have you looked it up in the OEIS? I just did. No relevant results.
    $endgroup$
    – Robert Soupe
    Jul 26 '18 at 18:19






  • 1




    $begingroup$
    Then I tried Select[Range[1000], PrimeQ[EulerPhi[#]^EulerPhi[EulerPhi[#]] + 1] &] in Wolfram Alpha. I probably need Mathematica if I want to confirm your assertion up to $10^4$.
    $endgroup$
    – Robert Soupe
    Jul 26 '18 at 18:22






  • 1




    $begingroup$
    @RobertSoupe : Pari/GP is a free software which is able to deal with that problem nicely. Note btw, that many $n$ have the same value $varphi(n)$ and thus the same $text{isprime}(f(n)$ -result, so time consumption could be much reduced when you avoid multiple computation of the $f(n)$ at different $n$ leading to the same result.
    $endgroup$
    – Gottfried Helms
    Jul 26 '18 at 20:58














  • 1




    $begingroup$
    @paw88789 To get a prime , this must be the case.
    $endgroup$
    – Peter
    Jul 26 '18 at 16:22






  • 2




    $begingroup$
    @Joffan if $q$ is an odd prime factor of $n$ , then $a^n+1$ is divisible by $a^{n/q}+1$
    $endgroup$
    – Peter
    Jul 26 '18 at 16:24






  • 1




    $begingroup$
    Have you looked it up in the OEIS? I just did. No relevant results.
    $endgroup$
    – Robert Soupe
    Jul 26 '18 at 18:19






  • 1




    $begingroup$
    Then I tried Select[Range[1000], PrimeQ[EulerPhi[#]^EulerPhi[EulerPhi[#]] + 1] &] in Wolfram Alpha. I probably need Mathematica if I want to confirm your assertion up to $10^4$.
    $endgroup$
    – Robert Soupe
    Jul 26 '18 at 18:22






  • 1




    $begingroup$
    @RobertSoupe : Pari/GP is a free software which is able to deal with that problem nicely. Note btw, that many $n$ have the same value $varphi(n)$ and thus the same $text{isprime}(f(n)$ -result, so time consumption could be much reduced when you avoid multiple computation of the $f(n)$ at different $n$ leading to the same result.
    $endgroup$
    – Gottfried Helms
    Jul 26 '18 at 20:58








1




1




$begingroup$
@paw88789 To get a prime , this must be the case.
$endgroup$
– Peter
Jul 26 '18 at 16:22




$begingroup$
@paw88789 To get a prime , this must be the case.
$endgroup$
– Peter
Jul 26 '18 at 16:22




2




2




$begingroup$
@Joffan if $q$ is an odd prime factor of $n$ , then $a^n+1$ is divisible by $a^{n/q}+1$
$endgroup$
– Peter
Jul 26 '18 at 16:24




$begingroup$
@Joffan if $q$ is an odd prime factor of $n$ , then $a^n+1$ is divisible by $a^{n/q}+1$
$endgroup$
– Peter
Jul 26 '18 at 16:24




1




1




$begingroup$
Have you looked it up in the OEIS? I just did. No relevant results.
$endgroup$
– Robert Soupe
Jul 26 '18 at 18:19




$begingroup$
Have you looked it up in the OEIS? I just did. No relevant results.
$endgroup$
– Robert Soupe
Jul 26 '18 at 18:19




1




1




$begingroup$
Then I tried Select[Range[1000], PrimeQ[EulerPhi[#]^EulerPhi[EulerPhi[#]] + 1] &] in Wolfram Alpha. I probably need Mathematica if I want to confirm your assertion up to $10^4$.
$endgroup$
– Robert Soupe
Jul 26 '18 at 18:22




$begingroup$
Then I tried Select[Range[1000], PrimeQ[EulerPhi[#]^EulerPhi[EulerPhi[#]] + 1] &] in Wolfram Alpha. I probably need Mathematica if I want to confirm your assertion up to $10^4$.
$endgroup$
– Robert Soupe
Jul 26 '18 at 18:22




1




1




$begingroup$
@RobertSoupe : Pari/GP is a free software which is able to deal with that problem nicely. Note btw, that many $n$ have the same value $varphi(n)$ and thus the same $text{isprime}(f(n)$ -result, so time consumption could be much reduced when you avoid multiple computation of the $f(n)$ at different $n$ leading to the same result.
$endgroup$
– Gottfried Helms
Jul 26 '18 at 20:58




$begingroup$
@RobertSoupe : Pari/GP is a free software which is able to deal with that problem nicely. Note btw, that many $n$ have the same value $varphi(n)$ and thus the same $text{isprime}(f(n)$ -result, so time consumption could be much reduced when you avoid multiple computation of the $f(n)$ at different $n$ leading to the same result.
$endgroup$
– Gottfried Helms
Jul 26 '18 at 20:58










1 Answer
1






active

oldest

votes


















0












$begingroup$

This is mostly just a summary of what I have found that is to big for a comment.



Since $varphi(varphi(n))$ must be a power of $2$, $varphi(n)=2^m p_1 p_2 p_3...p_m$. Where each of the $p_i$ is a distinct Fermat prime. Thus we have
$$varphi(n)^{varphi(varphi(n))}+1=(2^mp_1p_2p_3...p_m)^{2^r}+1.tag{1}$$
We also have that
$$n=2^uq_1q_2...q_sp_1^{e_1}p_2^{e_2}...p_m^{e_m}$$
where the $q_i$ are primes of the form $2^dp_i+1$, and the $e_i$ are each $0,1$ or $2$. If a $q_i$ is present in the factorization, then $e_i$ is at most $1$.



If we want to answer your question, it will probably be easiest to work with the right side of $(1)$.






share|cite|improve this answer









$endgroup$













    Your Answer





    StackExchange.ifUsing("editor", function () {
    return StackExchange.using("mathjaxEditing", function () {
    StackExchange.MarkdownEditor.creationCallbacks.add(function (editor, postfix) {
    StackExchange.mathjaxEditing.prepareWmdForMathJax(editor, postfix, [["$", "$"], ["\\(","\\)"]]);
    });
    });
    }, "mathjax-editing");

    StackExchange.ready(function() {
    var channelOptions = {
    tags: "".split(" "),
    id: "69"
    };
    initTagRenderer("".split(" "), "".split(" "), channelOptions);

    StackExchange.using("externalEditor", function() {
    // Have to fire editor after snippets, if snippets enabled
    if (StackExchange.settings.snippets.snippetsEnabled) {
    StackExchange.using("snippets", function() {
    createEditor();
    });
    }
    else {
    createEditor();
    }
    });

    function createEditor() {
    StackExchange.prepareEditor({
    heartbeatType: 'answer',
    autoActivateHeartbeat: false,
    convertImagesToLinks: true,
    noModals: true,
    showLowRepImageUploadWarning: true,
    reputationToPostImages: 10,
    bindNavPrevention: true,
    postfix: "",
    imageUploader: {
    brandingHtml: "Powered by u003ca class="icon-imgur-white" href="https://imgur.com/"u003eu003c/au003e",
    contentPolicyHtml: "User contributions licensed under u003ca href="https://creativecommons.org/licenses/by-sa/3.0/"u003ecc by-sa 3.0 with attribution requiredu003c/au003e u003ca href="https://stackoverflow.com/legal/content-policy"u003e(content policy)u003c/au003e",
    allowUrls: true
    },
    noCode: true, onDemand: true,
    discardSelector: ".discard-answer"
    ,immediatelyShowMarkdownHelp:true
    });


    }
    });














    draft saved

    draft discarded


















    StackExchange.ready(
    function () {
    StackExchange.openid.initPostLogin('.new-post-login', 'https%3a%2f%2fmath.stackexchange.com%2fquestions%2f2863551%2fare-there-further-primes-of-the-form-varphin-varphi-varphin1%23new-answer', 'question_page');
    }
    );

    Post as a guest















    Required, but never shown

























    1 Answer
    1






    active

    oldest

    votes








    1 Answer
    1






    active

    oldest

    votes









    active

    oldest

    votes






    active

    oldest

    votes









    0












    $begingroup$

    This is mostly just a summary of what I have found that is to big for a comment.



    Since $varphi(varphi(n))$ must be a power of $2$, $varphi(n)=2^m p_1 p_2 p_3...p_m$. Where each of the $p_i$ is a distinct Fermat prime. Thus we have
    $$varphi(n)^{varphi(varphi(n))}+1=(2^mp_1p_2p_3...p_m)^{2^r}+1.tag{1}$$
    We also have that
    $$n=2^uq_1q_2...q_sp_1^{e_1}p_2^{e_2}...p_m^{e_m}$$
    where the $q_i$ are primes of the form $2^dp_i+1$, and the $e_i$ are each $0,1$ or $2$. If a $q_i$ is present in the factorization, then $e_i$ is at most $1$.



    If we want to answer your question, it will probably be easiest to work with the right side of $(1)$.






    share|cite|improve this answer









    $endgroup$


















      0












      $begingroup$

      This is mostly just a summary of what I have found that is to big for a comment.



      Since $varphi(varphi(n))$ must be a power of $2$, $varphi(n)=2^m p_1 p_2 p_3...p_m$. Where each of the $p_i$ is a distinct Fermat prime. Thus we have
      $$varphi(n)^{varphi(varphi(n))}+1=(2^mp_1p_2p_3...p_m)^{2^r}+1.tag{1}$$
      We also have that
      $$n=2^uq_1q_2...q_sp_1^{e_1}p_2^{e_2}...p_m^{e_m}$$
      where the $q_i$ are primes of the form $2^dp_i+1$, and the $e_i$ are each $0,1$ or $2$. If a $q_i$ is present in the factorization, then $e_i$ is at most $1$.



      If we want to answer your question, it will probably be easiest to work with the right side of $(1)$.






      share|cite|improve this answer









      $endgroup$
















        0












        0








        0





        $begingroup$

        This is mostly just a summary of what I have found that is to big for a comment.



        Since $varphi(varphi(n))$ must be a power of $2$, $varphi(n)=2^m p_1 p_2 p_3...p_m$. Where each of the $p_i$ is a distinct Fermat prime. Thus we have
        $$varphi(n)^{varphi(varphi(n))}+1=(2^mp_1p_2p_3...p_m)^{2^r}+1.tag{1}$$
        We also have that
        $$n=2^uq_1q_2...q_sp_1^{e_1}p_2^{e_2}...p_m^{e_m}$$
        where the $q_i$ are primes of the form $2^dp_i+1$, and the $e_i$ are each $0,1$ or $2$. If a $q_i$ is present in the factorization, then $e_i$ is at most $1$.



        If we want to answer your question, it will probably be easiest to work with the right side of $(1)$.






        share|cite|improve this answer









        $endgroup$



        This is mostly just a summary of what I have found that is to big for a comment.



        Since $varphi(varphi(n))$ must be a power of $2$, $varphi(n)=2^m p_1 p_2 p_3...p_m$. Where each of the $p_i$ is a distinct Fermat prime. Thus we have
        $$varphi(n)^{varphi(varphi(n))}+1=(2^mp_1p_2p_3...p_m)^{2^r}+1.tag{1}$$
        We also have that
        $$n=2^uq_1q_2...q_sp_1^{e_1}p_2^{e_2}...p_m^{e_m}$$
        where the $q_i$ are primes of the form $2^dp_i+1$, and the $e_i$ are each $0,1$ or $2$. If a $q_i$ is present in the factorization, then $e_i$ is at most $1$.



        If we want to answer your question, it will probably be easiest to work with the right side of $(1)$.







        share|cite|improve this answer












        share|cite|improve this answer



        share|cite|improve this answer










        answered Jan 8 at 21:37









        coDE_RPcoDE_RP

        599




        599






























            draft saved

            draft discarded




















































            Thanks for contributing an answer to Mathematics Stack Exchange!


            • Please be sure to answer the question. Provide details and share your research!

            But avoid



            • Asking for help, clarification, or responding to other answers.

            • Making statements based on opinion; back them up with references or personal experience.


            Use MathJax to format equations. MathJax reference.


            To learn more, see our tips on writing great answers.




            draft saved


            draft discarded














            StackExchange.ready(
            function () {
            StackExchange.openid.initPostLogin('.new-post-login', 'https%3a%2f%2fmath.stackexchange.com%2fquestions%2f2863551%2fare-there-further-primes-of-the-form-varphin-varphi-varphin1%23new-answer', 'question_page');
            }
            );

            Post as a guest















            Required, but never shown





















































            Required, but never shown














            Required, but never shown












            Required, but never shown







            Required, but never shown

































            Required, but never shown














            Required, but never shown












            Required, but never shown







            Required, but never shown







            Popular posts from this blog

            Can a sorcerer learn a 5th-level spell early by creating spell slots using the Font of Magic feature?

            Does disintegrating a polymorphed enemy still kill it after the 2018 errata?

            A Topological Invariant for $pi_3(U(n))$